5

Let $a$, $b$ and $c$ be non-negative numbers such that $ab+ac+bc\neq0$. Prove that: $$ \dfrac{1}{\sqrt{2a^2+5ab+2b^2}}+\dfrac{1}{\sqrt{2b^2+5bc+2c^2}}+\dfrac{1}{\sqrt{2c^2+5ca+2a^2}} \geq\sqrt{\frac{3}{ab+ac+bc}}.$$

I solved this problem by Hölder: $$\left(\sum_{cyc}\dfrac{1}{\sqrt{2a^2+5ab+2b^2}}\right)^2\sum_{cyc}\frac{(a+b)^3}{(2a^2+5ab+2b^2)^2}\geq\left(\sum_{cyc}\frac{a+b}{2a^2+5ab+2b^2}\right)^3$$ and it remains to prove that $$(ab+ac+bc)\left(\sum_{cyc}\frac{a+b}{2a^2+5ab+2b^2}\right)^3\geq3\sum_{cyc}\frac{(a+b)^3}{(2a^2+5ab+2b^2)^2},$$ which is true by BW and by using computer.

In this topic https://artofproblemsolving.com/community/c6h542992 there is a proof (from gxggs), but it's very very complicated.

I found another way, a smooth enough, but it's still a very hard solution.

I am looking for a nice proof by hand, for which there is a possibility to find this proof during a competition.

Thank you!

Falrach
  • 4,242

4 Answers4

5

Remark: I give a proof using the so-called isolated fudging.

It suffices to prove that $$\frac{\sqrt{\frac{ab + bc + ca}{3}}}{\sqrt{2a^2 + 5ab + 2b^2}}\ge \frac{8c^2 + 9(a + b)c + 8ab}{8(a^2 + b^2 + c^2) + 26(ab + bc + ca)}. \tag{1}$$ Note: Taking cyclic sum on (1), we get the desired inequality.

If $c = 0$, it is easy.

If $c > 0$, WLOG, assume that $c = 1$. Let $p = a + b, ~ q = ab$. Then $0 \le q \le p^2/4$. It suffices to prove that $$\frac{\sqrt{(q + p)/3}}{\sqrt{2(p^2 - 2q) + 5q}} \ge \frac{8 + 9p + 8q}{8(p^2 - 2q + 1) + 26(q + p)}.$$ Squaring both sides, it suffices to prove that \begin{align*} &-92\,{q}^{3}+ \left( -224\,{p}^{2}+188\,p-224 \right) {q}^{2}+ \left( 64\,{p}^{4}-288\,{p}^{3}+313\,{p}^{2}+144\,p-128 \right) q \\ &\quad +64\,{p}^{5} -70\,{p}^{4}-60\,{p}^{3}+32\,{p}^{2}+64\,p \ge 0. \end{align*} Denote LHS by $f(q)$. We have $f''(q) = - 448p^2 + 376p - 448 - 552q$. So, we have $f''(q) < 0$ on $q \ge 0$. So, $f(q)$ is concave on $q \ge 0$. Also, we have $f(0) = 64p^5 - 70p^4 - 60p^3 + 32p^2 + 64p \ge 0$ and $f(p^2/4) = \frac{1}{16}\,p \left( 9\,{p}^{3}+96\,{p}^{2}+256\,p+256 \right) \left( p-2 \right) ^{2} \ge 0$. Thus, $f(q) \ge 0$ for all $q\in [0, p^2/4]$.

We are done.

River Li
  • 49,125
  • Thank you! I have found a very easy proof by $uvw$. – Michael Rozenberg Sep 13 '21 at 02:03
  • @MichaelRozenberg Nice. uvw is very powerful. – River Li Sep 13 '21 at 02:09
  • I have seen this technique a few times now and wondering if you have a general technique to find the lower bounds that sum up to $1.$ – dezdichado Sep 13 '21 at 02:33
  • 1
    @dezdichado If quadratic fraction actually work i.e. $ f(a,b,c)\ge\frac{p_1a^2+p_2b^2+p_3c^2+p_4ab+p_5bc+p_6ca}{a^2+b^2+c^2+q(ab+bc+ca)}$, we need to find the coefficients which satisfies such as $f(a_0, b_0, c_0)$ ($a_0, b_0, c_0$ is the equality case for the inequality $f(a,b,c) + f(b,c,a) + f(c,a,b) \ge 1$, also the stationary condition $f'(a_0, b_0, c_0) = 0$, also sum to 1. – River Li Sep 13 '21 at 03:09
  • @dezdichado If $f(a, b, c)$ is symmetric about $a, b$, we can simplify something such as in this proof, $p_1 = p_2$, $p_5 = p_6$, etc. – River Li Sep 13 '21 at 03:11
  • @RiverLi, yeah I can see that once you know something works. More mysterious question to me is to figure out what type needed. I think I have even seen some Pade approximants somewhere and also some rational functions with radicals like $\sqrt[3][]$ stuff in the denominator. – dezdichado Sep 13 '21 at 03:51
  • @dezdichado A unusual isolated fudging example is given by KaiRain@AoPS, see Baby LE's answer (Eq. (1) in particular): https://math.stackexchange.com/questions/336367/prove-sqrt3a-b3-sqrt3b-c3-sqrt3c-a3-ge-6 – River Li Sep 13 '21 at 03:58
  • @dezdichado Another nice isolated fudging example is given by math90@AoPS, that is $\frac{1}{2a+1}+\frac{a}{2a+bc}\ge\frac{4}{a+bc+4}$ (taking cyclic sum on it, we get the desired inequality $\frac{4}{a+bc+4}+\frac{4}{b+ac+4}+\frac{4}{c+ab+4}\leq1+\frac{1}{2a+1}+\frac{1}{2b+1}+\frac{1}{2c+1}$), see https://artofproblemsolving.com/community/c6h625375p3748344 – River Li Sep 13 '21 at 04:02
2

My attempt for this very hard inequality as conjecture :

Conjecture :

Let $x\geq z$ and $z\leq y\leq 1.5z$ and $z\geq 1$ then it seems we have :

$$0\le f\left(x\right)+f\left(y\right)-2f\left(\frac{2}{\frac{1}{x}+\frac{1}{y}}\right)$$

Where : $$f\left(a\right)=\frac{1}{\sqrt{2a^{2}+z\left(a-z\right)}}$$

The trick here is to use $a+b=u$ and $ab=b(u-b)$

Next it seems easier to show for $x,b,c>0$:

$$g\left(x\right)=\frac{2}{\sqrt{2\left(\frac{2}{\frac{1}{x+b}+\frac{1}{b+c}}\right)^{2}+b\left(\frac{2}{\frac{1}{x+b}+\frac{1}{b+c}}-b\right)}}+\frac{1}{\sqrt{2x^{2}+2c^{2}+5xc}}-\sqrt{\frac{3}{xb+bc+cx}}\geq 0$$

Hope it helps !

Barackouda
  • 3,879
1

Proof.

By Holder, we will prove $$\frac{(a^2+b^2+c^2+9)^3}{\sum\limits_{cyc}{(a^2+3)^3(2b^2+5bc+2c^2)} }\ge 1.$$ After homogenization, it is $$(ab+bc+ca)\left[(a+b+c)^2+ab+bc+ca\right]^3 \ge 3\sum_{\mathrm{cyc}}{(a+b)^3(a+c)^3(2b^2+5bc+2c^2)}.\tag{*}$$Equality holds at $(t,t,t), t>0$ or $abc=0.$

Let $a+b+c=3u; ab+bc+ca=3v^2; abc=w^3$. $(*)$ turns out $$f(w^3)=A(u,v^2)w^6+B(u,v^2)w^3+C(u,v^2)+27v^2(3u^2+v^2)^3\ge 0,\tag{**}$$where $f''(w^3)<0$ and by $uvw$ we need to prove the problem is true when $w^3$ achieve extremal value. Consider two cases:

  • $w^3=0.$
  • Two equal variables.

The desired result follows.


Indeed, we can demonstrate $RHS_{(*)}$ as \begin{align*} &2\left[(a+b)(b+c)(c+a)\right]^2.\sum_{cyc}(a+b)(a+c)+\sum_{cyc}bc(a+b)^3(a+c)^3\\&=2(9uv^2-w^3)^2(9u^2+3v^2)+w^3(a^5+b^5+c^5)+81v^8+9v^2w^3(a^3+b^3+c^3)+81uv^4w^3, \end{align*} Notice that: $a^3+b^3+c^3=27u^3-27uv^2+3w^3; a^5 + b^5 + c^5 = 243u^5-405u^3v^2+135uv^4+5(9u^2-3v^2)w^3.$

Now, with the form $(**)$ we obtain $$\color{blue}{A=-\left[2(9u^2+3v^2)+5(9u^2-3v^2)+27v^2 \right]=-\left(63u^2+18v^2\right)<0.}$$ Hence, $f(r)$ is concave and we prove in two cases.

  • In case $w^3=0,$ $(*)$ becomes $$bc(b^2+3bc+c^2)^3\ge 3\left[(bc)^3(2b^2+5bc+2c^2)+2c^2b^3(b+c)^3+2b^2c^3(b+c)^3\right],$$ $$\iff (b^2+3bc+c^2)^3\ge 3\left[(bc)^2(2b^2+5bc+2c^2)+2cb^2(b+c)^3+2bc^2(b+c)^3\right]. \tag{1}$$

If $bc=0,$ then $(1)$ is true.

If $bc\neq 0,$ then let $b=1$ and $(1)$ turns out $$(1+3c+c^2)^3\ge 3\left[c^2(2+5c+2c^2)+2c(1+c)^3+2c^2(1+c)^3\right],$$

$$\iff c^6 + 3 c^5 - 6 c^3 + 3 c + 1\ge 0,$$ $$\iff (c^3-1)^2+c[3(c^2-1)^2+2c^2]\ge 0.$$

  • In case two equal variables

Since this inequality is homogeneous and for $b=c=0$ it's obviously true, we can assume $b=c=1$, which gives $$(2a+1)\left[(a+2)^2+2a+1\right]^3 \ge 3\left[9(a+1)^6+16(a+1)^3(2a^2+5a+2)\right],$$ $$\iff 2 a^7 + 10 a^6 + 6 a^5 - 18 a^4 - 18 a^3 + 6 a^2 + 10 a + 2 \ge 0,$$ $$\iff 2(a-1)^2(a+1)^3(a^2+4a+1)\ge 0.$$

I hope to see nice proof for $(*).$

Generally, for all $n \ge \dfrac{10-3\sqrt{3}}{2},$ the following inequality $$\boxed{\dfrac{1}{\sqrt{a^2+nab+b^2}}+\dfrac{1}{\sqrt{b^2+nbc+c^2}}+\dfrac{1}{\sqrt{c^2+nca+a^2}} \ge 3\sqrt{\dfrac{3}{(n+2)(ab+bc+ca)}},}$$ holds for $a,b,c\ge 0: ab+bc+ca>0.$

At $\color{blue}{n=\dfrac{10-3\sqrt{3}}{2},}$ equality holds also at $(0,t,t)$ ($t>0$)

0

I found the following solution.

We need to prove that: $$\sum_{cyc}\left(\frac{1}{\sqrt{2a^2+5ab+2b^2}}-\frac{1}{\sqrt{3(ab+ac+bc)}}\right)\geq0$$ or $$\sum_{cyc}\frac{3ac+3bc-2a^2-2b^2-2ab}{2a^2+5ab+2b^2+\sqrt{3(ab+ac+bc)(2a^2+5ab+2b^2)}}\geq0$$ or $$\sum_{cyc}\frac{(c-a)(2a+b)-(b-c)(2b+a)}{2a^2+5ab+2b^2+\sqrt{3(ab+ac+bc)(2a^2+5ab+2b^2)}}\geq0$$ or $$\sum_{cyc}(a-b)\left(\tfrac{2b+c}{2b^2+5bc+2c^2+\sqrt{3(ab+ac+bc)(2b^2+5bc+2c^2)}}-\tfrac{2a+c}{2a^2+5ac+2c^2+\sqrt{3(ab+ac+bc)(2a^2+5ac+2c^2)}}\right)\geq0$$ or $$\sum_{cyc}(a-b)^2\left((2a+c)(2b+c)-\tfrac{3c\sqrt{3(ab+ac+bc)(2a+c)(2b+c)}}{\sqrt{(a+2c)(2b+c)}+\sqrt{(2a+c)(b+2c)}}\right)\left(2a^2+5ab+2b^2+\sqrt{3(ab+ac+bc)(2a^2+5ab+2b^2)}\right)\geq0.$$ Now, let $a\geq b\geq c$.

Thus, since $$(2a+c)(2b+c)-\tfrac{3c\sqrt{3(ab+ac+bc)(2a+c)(2b+c)}}{\sqrt{(a+2c)(2b+c)}+\sqrt{(2a+c)(b+2c)}}\geq0,$$ $$(2a+b)(2c+b)-\tfrac{3b\sqrt{3(ab+ac+bc)(2a+b)(2c+b)}}{\sqrt{(a+2b)(2c+b)}+\sqrt{(2a+b)(c+2b)}}\geq0$$ and $$b(a-c)\geq a(b-c),$$ it remains to prove that: $$a^2\left((2a+b)(2c+b)-\tfrac{3b\sqrt{3(ab+ac+bc)(2a+b)(2c+b)}}{\sqrt{(a+2b)(2c+b)}+\sqrt{(2a+b)(c+2b)}}\right)\left(2a^2+5ac+2c^2+\sqrt{3(ab+ac+bc)(2a^2+5ac+2c^2)}\right)+$$ $$+b^2\left((2b+a)(2c+a)-\tfrac{3a\sqrt{3(ab+ac+bc)(2b+a)(2c+a)}}{\sqrt{(b+2a)(2c+a)}+\sqrt{(2a+b)(c+2a)}}\right)\left(2b^2+5bc+2c^2+\sqrt{3(ab+ac+bc)(2b^2+5bc+2c^2)}\right)\geq0.$$ But $$2a^2+5ac+2c^2+\sqrt{3(ab+ac+bc)(2a^2+5ac+2c^2)}\geq2b^2+5bc+2c^2+\sqrt{3(ab+ac+bc)(2b^2+5bc+2c^2)},$$ $$\sqrt{(a+2b)(2c+b)}+\sqrt{(2a+b)(c+2b)}=\sqrt{4b^2+5ab+4ac+5bc+2\sqrt{(2a^2+5ab+2b^2)(2c^2+5bc+2b^2)}}\geq$$ $$\geq\sqrt{4b^2+4ab+4ac+4bc+2\sqrt{(2a^2+4ab+2b^2)(2c^2+4bc+2b^2)}}=$$ $$=\sqrt{8(a+b)(b+c)}$$ and similarly $$\sqrt{(b+2a)(2c+a)}+\sqrt{(2a+b)(c+2a)}\geq\sqrt{8(a+b)(a+c)}.$$ Thus, it's enough to prove that: $$a^2(2a+b)(2c+b)+b^2(2b+a)(2c+a)\geq\tfrac{3ab\sqrt{3(ab+ac+bc)}}{\sqrt{8(a+b)}}\left(a\sqrt{\tfrac{2a+b)(2c+b)}{b+c}}+b\sqrt{\tfrac{(2b+a)(2c+a)}{a+c}}\right).$$ Now, by C-S $$a\sqrt{\tfrac{2a+b)(2c+b)}{b+c}}+b\sqrt{\tfrac{(2b+a)(2c+a)}{a+c}}\leq\sqrt{(a+b)\left(\tfrac{a(2a+b)(2c+b)}{b+c}+\tfrac{b(2b+a)(2c+a)}{a+c}\right)}\leq$$ $$\leq\sqrt{(a+b)\left(\tfrac{a(2a+b)\frac{3}{2}(c+b)}{b+c}+\tfrac{b(2b+a)\frac{3}{2}(c+a)}{a+c}\right)}=\sqrt{3(a+b)(a^2+ab+b^2)}.$$ Id est, it's enough to prove that: $$8\left(a^2(2a+b)(2c+b)+b^2(2b+a)(2c+a)\right)^2\geq81a^2b^2(ab+ac+bc)(a^2+ab+b^2),$$ which is true because $$8\left(a^2(2a+b)(2c+b)+b^2(2b+a)(2c+a)\right)^2=$$ $$=8\left(2a^2b^2+2a^3b+2ab^3+2a^2bc+2b^2ac+4a^3c+4b^3c\right)^2\geq$$ $$\geq8\left(2a^2b^2+2a^3b+2ab^3+2a^2bc+2b^2ac+4a^2bc+4b^2ac\right)^2=$$ $$=32a^2b^2(a^2+ab+b^2+3ac+3bc)^2\geq$$ $$\geq32a^2b^2((a^2+ab+b^2)^2+6(a^2+ab+b^2)(ac+bc))=$$ $$=32a^2b^2(a^2+ab+b^2)(a^2+ab+b^2+6(ac+bc))\geq$$ $$\geq32a^2b^2(a^2+ab+b^2)(3ab+3(ac+bc))=$$ $$=96a^2b^2(a^2+ab+b^2)(ab+ac+bc)\geq$$ $$\geq81a^2b^2(a^2+ab+b^2)(ab+ac+bc)$$ and we are done!


I think, the best way it's the following.

By the gxggs's idea (see here https://artofproblemsolving.com/community/c6h542992p3142759 ) it's enough to prove that $$\sum_{cyc}{\frac{12(a+b)}{17a^2+38ab+17b^2}}\geq \sqrt{\frac{3}{ab+bc+ca}}.$$ Now, let $a+b+c=3u$, $ab+ac+bc=3v^2$ and $abc=w^3$.

Thus, since for $(a,b,c)=(1,\zeta,\zeta^2)$, where $\zeta^2+\zeta+1=0$, we have $w^3\neq0$, $$u=v^2=a^2+ab+b^2=a^2+ac+c^2=b^2+bc+c^2=0$$ and $$\prod_{cyc}(17a^2+38ab+17b^2)=21^3w^6,$$ we need to prove that $f(w^3)\geq0,$ where $f$ is a concave function,

which by $uvw$ (about $uvw$ see here: https://artofproblemsolving.com/community/c6h278791 ) says that it's enough to prove $$\sum_{cyc}{\frac{12(a+b)}{17a^2+38ab+17b^2}}\geq \sqrt{\frac{3}{ab+bc+ca}}$$ in the following cases.

  1. Two variables are equal (it's enough to assume $b=c=1$);

  2. $w^3=0$ (it's enough to assume $c=0$ and $b=1$),

which easy to check.

  • 2
    You may separate two parts using
    before "I think, the best way it's the following." Or, move the uvw solution to another one.
    – River Li Sep 18 '21 at 02:38